Menu Close

2014-x-3-4029x-2-2-0-x-1-lt-x-2-lt-x-3-x-2-x-1-x-3-




Question Number 64124 by ANTARES VY last updated on 13/Jul/19
(√(2014))x^3 −4029x^2 +2=0  x_1 <x_2 <x_3   x_2 (x_1 +x_3 )=?
$$\sqrt{\mathrm{2014}}\boldsymbol{\mathrm{x}}^{\mathrm{3}} −\mathrm{4029}\boldsymbol{\mathrm{x}}^{\mathrm{2}} +\mathrm{2}=\mathrm{0} \\ $$$$\boldsymbol{\mathrm{x}}_{\mathrm{1}} <\boldsymbol{\mathrm{x}}_{\mathrm{2}} <\boldsymbol{\mathrm{x}}_{\mathrm{3}} \\ $$$$\boldsymbol{\mathrm{x}}_{\mathrm{2}} \left(\boldsymbol{\mathrm{x}}_{\mathrm{1}} +\boldsymbol{\mathrm{x}}_{\mathrm{3}} \right)=? \\ $$

Leave a Reply

Your email address will not be published. Required fields are marked *